2022 AMC 12A Problems/Problem 18

Revision as of 21:11, 11 November 2022 by Sugar rush (talk | contribs)

Problem

Let $T_k$ be the transformation of the coordinate plane that first rotates the plane $k$ degrees counter-clockwise around the origin and then reflects the plane across the $y$-axis. What is the least positive integer $n$ such that performing the sequence of transformations $T_1, T_2, T_3, \cdots, T_n$ returns the point $(1,0)$ back to itself?

Solution

Let $A_{n}$ be the point $(\cos n^{\circ}, \sin n^{\circ})$.

Starting with $n=0$, the sequence goes \[A_{0}\rightarrow A_{179}\rightarrow A_{359}\rightarrow A_{178}\rightarrow A_{358}\rightarrow A_{177}\rightarrow A_{357}\rightarrow\cdots\]

We see that it takes $2$ turns to downgrade the point by $1^{\circ}$. Since the fifth point in the sequence is $A_{177}$, the answer is $5+2(177)=\boxed{\textbf{(A)}~359}$

Video Solution by Professor Chen Education Palace

https://youtu.be/QQrsKTErJn8

See also

2022 AMC 12A (ProblemsAnswer KeyResources)
Preceded by
Problem 17
Followed by
Problem 19
1 2 3 4 5 6 7 8 9 10 11 12 13 14 15 16 17 18 19 20 21 22 23 24 25
All AMC 12 Problems and Solutions
2022 AMC 10A (ProblemsAnswer KeyResources)
Preceded by
Problem 17
Followed by
Problem 19
1 2 3 4 5 6 7 8 9 10 11 12 13 14 15 16 17 18 19 20 21 22 23 24 25
All AMC 10 Problems and Solutions

The problems on this page are copyrighted by the Mathematical Association of America's American Mathematics Competitions. AMC logo.png